· 2017. 12. 6. · options: 1 ) erudite 2 ) retard 3 ) disuse 4 ) altered correct answer: retard...

44
CANDIDATE NAME : WWW.kvclasses.com EXAM START TIME : 14:45:00 EXAM DURATION : 02:00 Hrs TOTAL NO OF QUESTIONS : 200 Click to Print logout Challenges REGISTRATION NUMBER : WWW.kvclasses.com EXAM DATE : 25-November-2017 EXAM NAME : SSC Examination TOTAL MARKS : 200 QID : 601 - Select the related word from the given alternatives. Thirsty : Water :: Hungry : ? दए गए वकप से संबंधत शद को चुनए। यासा : पानी :: भूखा : ? Options: 1) Fasting उपवास 2) Food खा 3) Diet आहार 4) Poor गरीब Correct Answer: Food खा Candidate Answer: Food खा QID : 602 - Select the related letters from the given alternatives. BDF : ZBD :: CEG : ? दए गए वकप से संबंधत अर को चुनए। BDF : ZBD :: CEG : ? Options: 1) QST 2) YZB 3) ACE 4) GEC Correct Answer: ACE Candidate Answer: ACE QID : 603 - Select the related number from the given alternatives. 9 : 729 :: 3 : ? दए गए वकप से संबंधत संया को चुनए। 9 : 729 :: 3 : ? Options: 1)9 2) 81 3) 27 4) 273 Correct Answer: 27 Candidate Answer: 27 QID : 604 - Select the odd word from the given alternatives. दए गए वकप से वषम शद को चुनए। Downloaded from www.kvclasses.com Visit www.kvclasses.com for tons of free study materials and latest exam updates. www.kvclasses.com

Upload: others

Post on 28-Jan-2021

2 views

Category:

Documents


0 download

TRANSCRIPT

  • CANDIDATE NAME : WWW.kvclasses.com

    EXAM START TIME : 14:45:00 EXAM DURATION  :  02:00 Hrs

    TOTAL NO OF QUESTIONS :  200

    Click to Print logout Challenges

    REGISTRATION NUMBER : WWW.kvclasses.comEXAM DATE : 25-November-2017 EXAMNAME : SSC ExaminationTOTAL MARKS : 200

    QID : 601 - Select the related word from the given alternatives.

    Thirsty : Water :: Hungry : ?

    �दए गए �वक�प� म� स ेसंबं�धत श�द को चु�नए।

    �यासा : पानी :: भूखा : ?Options:1) Fastingउपवास2) Foodखा�3) Dietआहार4) Poorगरीब

    Correct Answer: Foodखा�

    Candidate Answer: Foodखा�

    QID : 602 - Select the related letters from the given alternatives.

    BDF : ZBD :: CEG : ?

    �दए गए �वक�प� म� स ेसंबं�धत अ�र� को चु�नए।

    BDF : ZBD :: CEG : ?Options:1) QST2) YZB3) ACE4) GECCorrect Answer: ACECandidate Answer: ACE

    QID : 603 - Select the related number from the given alternatives.

    9 : 729 :: 3 : ?

    �दए गए �वक�प� म� स ेसंबं�धत सं�या को चु�नए।

    9 : 729 :: 3 : ?Options:1) 92) 813) 274) 273Correct Answer: 27Candidate Answer: 27

    QID : 604 - Select the odd word from the given alternatives.

    �दए गए �वक�प� म� स े�वषम श�द को चु�नए।

    Downloaded from www.kvclasses.com

    Visit www.kvclasses.com for tons of free study materials and latest exam updates.

    www.

    kvcl

    asse

    s.com

    http://thepracticetest.in/cs_imd_17/logout.php?appid=1368062d74107fc05177712c90c3885chttp://ssconline.nic.in/challengesystem/sendrollno.jsp?rollno=2201025683P&scode=108&key=e6099c0ec5abd5111ddf96af6e38c53f

  • Options:1) Orangeनारंगी2) Peachआड�3) Onion�याज4) Pineappleअनानास

    Correct Answer: Onion�याज

    Candidate Answer: Onion�याज

     QID : 605 - Select the odd letters from the given alternatives.

    �दए गए �वक�प� म� स े�वषम अ�र� को चु�नए।Options:1) PSV2) DFH3) JLN4) TVXCorrect Answer: PSVCandidate Answer: PSV QID : 606 - Select the odd number from the given alternatives.

    �दए गए �वक�प� म� स े�वषम सं�या को चु�नए।Options:1) 2872) 3013) 3274) 371Correct Answer: 327Candidate Answer: 327 QID : 607 - A series is given, with one word missing. Choose the correct alternative from the given ones that will complete the series.

    Dreadful, Ideal, Ladies, Leader, Sturdy, ?

    एक अनु�म �दया गया ह,ै �जसम� एक श�द लु�त है। �दए गए �वक�प� म� स ेवह सही �वक�प चु�नए जो अनु�म को पूरा करे।

    Dreadful, Ideal, Ladies, Leader, Sturdy, ?Options:1) Erudite

    2) Retard

    3) Disuse

    4) AlteredCorrect Answer: RetardCandidate Answer: Retard QID : 608 - A series is given, with one term missing. Choose the correct alternative from the given ones that will complete the series.

    S, N, J, G, E ?

    एक अनु�म �दया गया ह,ै �जसम� एक पद लु�त है। �दए गए �वक�प� म� स ेवह सही �वक�प चु�नए जो अनु�म को पूरा करे।

    S, N, J, G, E ?Options:1) C

    2) B

    3) F

    4) DCorrect Answer: DCandidate Answer: D 

    Downloaded from www.kvclasses.com

    Visit www.kvclasses.com for tons of free study materials and latest exam updates.

    www.

    kvcl

    asse

    s.com

  • QID : 609 - A series is given, with one number missing. Choose the correct alternative from the given ones that will complete the series.

    0.01, 0.025, ?, 0.055, 0.07

    एक अनु�म �दया गया ह,ै �जसम� एक सं�या लु�त है। �दए गए �वक�प� म� स ेवह सही �वक�प चु�नए जो अनु�म को पूरा करे।

    0.01, 0.025, ?, 0.055, 0.07Options:1) 0.035

    2) 0.03

    3) 0.04

    4) 0.05Correct Answer: 0.04Candidate Answer: 0.04 

    QID : 610 - Rishita's birthday is on Monday 29th May. On what day of the week will be Ahana's Birthday in the same year, if Ahana was born on 2nd October?

    ऋ�षता का ज�म�दन 29 मई, �दन सोमवार को है। य�द अहाना का ज�म 2 अ�टूबर को �आ था, तो उसी वष� अहाना का ज�म�दन स�ताह के �कस �दन होगा?Options:1) Mondayसोमवार2) Wednesdayबुधवार3) Fridayशु�वार4) Thursdayगु�वार

    Correct Answer: Mondayसोमवार

    Candidate Answer: Mondayसोमवार

     QID : 611 - The weights of 4 boxes are 30, 70, 10 and 20 Kg. Which of the following cannot be the total weight, in Kg., of any combination of these boxes and in acombination a box can be used only once?

    चार ब�स� का वजन 30, 70, 10 और 20 �क.�ा. है। �न�न�ल�खत म� स ेब�स� के �कसी भी संयोजन स ेकुल वज़न, �क.�ा. म� कौन सा नह� हो सकता ह ैऔर एक संयोजन म� एक ब�सा एक ही बार �योग म� लाया जासकता ह?ैOptions:1) 1402) 1303) 1104) 100Correct Answer: 140Candidate Answer: 140 QID : 612 - From the given words, select the word which cannot be formed using the letters of the given word.

    OUTPACED

    �न�न�ल�खत �वक�प� म� स ेवह श�द चु�नए जो �दए गए श�द के अ�र� का �योग करके नह� बनाया जा सकता है।

    OUTPACEDOptions:1) TAPED2) ACUTE3) DICOT4) OPTEDCorrect Answer: DICOTCandidate Answer: DICOT QID : 613 - If STANDBY is coded as HGZMWYB, then how will DEW be coded as?

    य�द STANDBY को �कसी कोड म� HGZMWYB �लखा जाता ह,ै तो DEW को उसी कोड म� �या �लखा जाएगा?Options:1) NOL

    2) BGT

    3) DVW

    4) WVDCorrect Answer: WVDCandidate Answer: WVD 

    Downloaded from www.kvclasses.com

    Visit www.kvclasses.com for tons of free study materials and latest exam updates.

    www.

    kvcl

    asse

    s.com

  •  QID : 614 - In a certain code language, '+' represents '×', '-' represents '+', '×' represents '÷' and '÷' represents '-'. What is the answer to the following question?

    36 × 9 - 8 + 4 = ?

    �कसी कोड भाषा म�, '+' का मतलब '×' ह,ै '-' का मतलब '+' ह,ै '×' का मतलब '÷' ह ैऔर '÷' का मतलब '-' है। �न�न�ल�खत �� का उ�र �या ह?ै

    36 × 9 - 8 + 4 = ?Options:1) 36

    2) 79

    3) 3

    4) 8Correct Answer: 36Candidate Answer: 36 QID : 615 - If 56$28 = 21, 2$3 = 5, 9$15 = 15 then what is the value of 8$15 = ?

    य�द 56$28 = 21, 2$3 = 5, 9$15 = 15 तो 8$15 = ? का मान �या होगा?Options:1) 142) 963) 484) 13Correct Answer: 14Candidate Answer: 14 QID : 616 -

    Options:1) 8

    2) 1

    3) 14

    4) 10Correct Answer: 1Candidate Answer: 1 QID : 617 - A and B start running from the same point. A runs 13 km West, then turns South and runs 12 km, then turns to his right and runs 5 km. B runs 4 km Souththen turns to his right and runs 18 km. Where is B with respect to A now?

    A और B एक ही �ब�� स ेदौड़ना शु� करत ेह�। A, 13 �क.मी. प��म क� ओर दौड़ता ह,ै �फर द��ण क� ओर मुड़ जाता ह ैऔर 12 �क.मी. दौड़ता ह,ै �फर अपनी दा� ओर मुड़ जाता ह ैऔर 5 �क.मी. दौड़ता है। B, 4�क.मी. द��ण क� ओर दौड़ता ह,ै �फर अपनी दा� ओर मुड़ जाता ह ैऔर 18 �क.मी. दौड़ता है। A क� ��थ�त स ेअब B कहा ंपर ह?ैOptions:1) 8 km South8 �क.मी. द��ण2) 8 km North8 �क.मी. उ�र3) 16 km North16 �क.मी. उ�र4) 16 km South16 �क.मी. द��णCorrect Answer: 8 km North8 �क.मी. उ�र

    Downloaded from www.kvclasses.com

    Visit www.kvclasses.com for tons of free study materials and latest exam updates.

    www.

    kvcl

    asse

    s.com

  • Candidate Answer: 8 km North8 �क.मी. उ�र QID : 618 - In the question two statements are given, followed by two conclusions, I and II. You have to consider the statements to be true even if it seems to be atvariance from commonly known facts. You have to decide which of the given conclusions, if any, follows from the given statements.

    Statement1 : All pens are pencils.2 : Some pens are ballpens.

    ConclusionI : Some ballpens are pencils.II : All pencils are ballpens.

    �� म� दो कथन �दये गय ेह� �जसके आग ेदो �न�कष� I और II �नकाल ेगय ेह�। आपको मानना ह ै�क दोनो कथन स�य ह ैचाह ेवह सामा�यत: �ात त�य� स े�भ� �तीत होत ेह�। आपको �नण�य करना ह ैक� �दए गए�न�कष� म� स ेकौन-सा/कौन-स े�न��त �प स ेकथन �ारा सही �नकाला जा सकता ह/ैसकत ेह�, य�द कोई हो।

    कथन1 : सभी पेन प��सल ह�।2 : कुछ पेन बॉलपेन ह�।

    �न�कष�I : कुछ बॉलपेन प��सल ह�।II : सभी प��सल बॉलपेन ह�।Options:1) Only conclusion I followsकेवल �न�कष� I सही है2) Only conclusion II followsकेवल �न�कष� II सही है3) Both I and II followदोन� �न�कष� I और II सही है4) Neither I nor II followsना तो �न�कष� I सही ह ैना ही �न�कष� IICorrect Answer: Only conclusion I followsकेवल �न�कष� I सही हैCandidate Answer: Only conclusion I followsकेवल �न�कष� I सही है QID : 619 -

    Downloaded from www.kvclasses.com

    Visit www.kvclasses.com for tons of free study materials and latest exam updates.

    www.

    kvcl

    asse

    s.com

  • Options:1) 

    2) 

    3) 

    4) 

    Correct Answer:

    Candidate Answer:

     QID : 620 -

     

    Downloaded from www.kvclasses.com

    Visit www.kvclasses.com for tons of free study materials and latest exam updates.

    www.

    kvcl

    asse

    s.com

  • Options:1) E,H2) E,B3) D,H4) A,GCorrect Answer: E,HCandidate Answer: E,H QID : 621 -

    Options:1) 

    2) 

    3) 

    4) 

    Correct Answer:

    Candidate Answer:

     

    Downloaded from www.kvclasses.com

    Visit www.kvclasses.com for tons of free study materials and latest exam updates.

    www.

    kvcl

    asse

    s.com

  • QID : 622 -

    Options:1) 

    2) 

    3) 

    4) 

    Correct Answer:

    Candidate Answer:

     

    Downloaded from www.kvclasses.com

    Visit www.kvclasses.com for tons of free study materials and latest exam updates.

    www.

    kvcl

    asse

    s.com

  • QID : 623 -

    Options:1) 

    2) 

    3) 

    4) 

    Correct Answer:

    Candidate Answer:

     

    Downloaded from www.kvclasses.com

    Visit www.kvclasses.com for tons of free study materials and latest exam updates.

    www.

    kvcl

    asse

    s.com

  • QID : 624 -

    Options:1) 

    2) 

    3) 

    4) 

    Correct Answer:

    Downloaded from www.kvclasses.com

    Visit www.kvclasses.com for tons of free study materials and latest exam updates.

    www.

    kvcl

    asse

    s.com

  • Candidate Answer:

     QID : 625 -

    Options:1) 98,66,69,88

    2) 10,42,58,56

    3) 03,21,98,89

    4) 86,95,32,96Correct Answer: 86,95,32,96Candidate Answer: 86,95,32,96 QID : 626 - What is the LCM of 65 and 169?

    65 और 169 का लघु�म समापवत�क �या ह?ैOptions:1) 9452) 8453) 7454) 645Correct Answer: 845Candidate Answer: 845 

    Downloaded from www.kvclasses.com

    Visit www.kvclasses.com for tons of free study materials and latest exam updates.

    www.

    kvcl

    asse

    s.com

  • QID : 627 - A can paint a house in 18 days and B can do it in 36 days. Along with C, they did the job in 6 days only. Then in how many days, C alone can do the job?

    A 18 �दन� म� एक घर को प�ट कर सकता ह ैऔर B इस े36 �दन� म� कर सकता है। C के साथ, उ�ह�न ेकेवल 6 �दन� म� ही इस काम को पूरा �कया। तो, अकेल ेC इस काम को �कतन े�दन� म� कर सकता ह?ैOptions:1) 82) 93) 104) 12Correct Answer: 12Candidate Answer: 12 QID : 628 - What is the diameter (in cm) of a sphere of surface area 5544 sq cm?

    5544 वग� स�.मी. पृ�ीय �े�फल वाल ेगोल ेका �ास (स�.मी. म�) �या ह?ैOptions:1) 422) 213) 31.54) 63Correct Answer: 42Candidate Answer: 42 QID : 629 - A t-shirt of Rs 400 is offered at 25% discount and a shirt of Rs 800 at 40% discount. If Amit bought 2 t-shirts and 4 shirts, what is the effective discount (in%) he got?

    एक 400 � क� ट�-शट� 25% छूट पर बेची जाती और 800 � क� शट� 40% छूट पर बेची जाती है। य�द अ�मत न े2 ट�-शट� और 4 शट� खरीदे, तो उस े�कतनी �भावी छूट (% म�) �मली?Options:1) 372) 353) 334) 31Correct Answer: 37Candidate Answer: 37 QID : 630 - If A's wealth is 7/9 times of B's and C's is 4/3 times of B's, what is the ratio of C's wealth to A's?

    अगर A का धन B के धन स े7/9 गुना और C का धन B के धन स े4/3 गुना ह,ै तो C के धन का A के धन स े�या अनुपात ह?ैOptions:1) (7:12)2) (28:27)3) (27:28)4) (12:7)Correct Answer: (12:7)Candidate Answer: (12:7) QID : 631 - The average of six consecutive odd numbers is 30. What is the largest number?

    छह लगातार �वषम सं�या� का औसत 30 है। सबसे बड़ी सं�या �या ह?ैOptions:1) 352) 333) 374) 39Correct Answer: 35Candidate Answer: 35 QID : 632 - If a trader decides to sell ground nut oil at Rs 120/litre he faces a loss of 10%. Now if he decides to sell it at Rs 160/litre, what will be the profit (in %)?

    य�द एक �ापारी 120 � ��त लीटर के भाव स ेमूंगफली का तेल बेचन ेका फैसला करता ह ैतो उस े10% का नुकसान होता है। अब अगर वह इस े160 � ��त लीटर पर बेचन ेका फैसला करता ह ैतो इसका �कतनालाभ (% म�) होगा?Options:1) 302) 203) 104) 25Correct Answer: 20Candidate Answer: 20 QID : 633 - A man donates 60% of his wealth to charity and gives the rest to his children. Amount which the children gets is what percent of the amount which wasdonated?

    एक ��� अपनी संप�� का 60% दान कर देता ह ैऔर बाक� अपन ेब�च� को देता है। ब�चो को �मला धन दान �कय े�ए धन का �कतना ��तशत ह?ै

    Downloaded from www.kvclasses.com

    Visit www.kvclasses.com for tons of free study materials and latest exam updates.

    www.

    kvcl

    asse

    s.com

  • Options:1) 1502) 66.673) 404) 120Correct Answer: 66.67Candidate Answer: 66.67 QID : 634 - If a person jogs at 11 km/hr instead of 10 km/hr, he would have walked 500 meters more in the same time. The actual distance (in kms) jogged by him is?

    य�द कोई ��� 10 �कलोमीटर / घंटा क� बजाय 11 �कलोमीटर / घंटा क� ग�त स ेजॉग करता ह,ै तो वह उसी समय म� 500 मीटर अ�धक चलेगा। उसके �ारा जॉ�ग�ग क� गई वा�त�वक �री (�कलोमीटर म�) �कतनीह?ैOptions:1) 62) 73) 54) 8Correct Answer: 5Candidate Answer: 5 QID : 635 - Krish invested an amount of x rupees in a certain business offering 12% per annum for 1st year and 20% per annum for 2nd year and received an amountof Rs 60,480 after the two years. What is x (in Rs)?

    कृष न ेएक �न��त �वसाय म� x �पए क� रा�श को पहले वष� के �लए 12% ��त वष� क� दर स ेऔर �सर ेवष� के �लए 20% ��त वष� क� दर स े�नवेश �कया और दो साल बाद 60,480 � क� रा�श �ा�त क�। x (� म�)�या ह?ैOptions:1) 400002) 420003) 460004) 45000Correct Answer: 45000Candidate Answer: 45000 QID : 636 - If (9/5)(x/2 - 10/3) + 4x/5 = 7x/10, then what is the value of x?

    य�द (9/5)(x/2 - 10/3) + 4x/5 = 7x/10, तो x का मान �या ह?ैOptions:1) -62) 1/63) 64) -1/6Correct Answer: 6Candidate Answer: 6 

    QID : 637 - If a3 - b3 = 91 and ab = 30, then what is the value of a - b?

    य�द a3 - b3 = 91 और ab = 30 ह,ै तो a - b का मान �या ह?ैOptions:1) 12) 83) 44) 6Correct Answer: 1Candidate Answer: 1 QID : 638 - Thrice a fraction is greater than its reciprocal by 2/15. What is the fraction?

    एक �भ� का 3 गुना उसके �ु��म स े2/15 स ेअ�धक है। �भ� �या ह?ैOptions:1) 5/32) 3/43) 4/34) 3/5Correct Answer: 3/5Candidate Answer: 3/5 QID : 639 - What is the sum of the first 14 terms of an arithmetic progression if the first term is 25 and last term is -53?

    एक समांतर �ेणी के �थम 14 पद� का योग �या ह,ै य�द पहला पद 25 ह ैऔर अं�तम पद -53 है।Options:1) -1302) 943) 984) -196Correct Answer: -196

    Downloaded from www.kvclasses.com

    Visit www.kvclasses.com for tons of free study materials and latest exam updates.

    www.

    kvcl

    asse

    s.com

  • Candidate Answer: -196 QID : 640 - What is the reflection of the point (3, -2) in the line y = 1?

    रेखा y = 1 म� �ब�� (3, -2) का ��त�ब�ब �या ह?ैOptions:1) (3 , 4)2) (-1 , -2)3) (-3 , 4)4) (-1 , 2)Correct Answer: (3 , 4)Candidate Answer: (3 , 4) QID : 641 - The distance between the points (4,8) and (k,-4) is 15. What is the value of k?

    �ब�� (4, 8) और (k, -4) के बीच क� �री 15 है। k का मान �या ह?ैOptions:1) 52) 33) -34) -5Correct Answer: -5Candidate Answer: -5 QID : 642 - What is the equation of the line perpendicular to the line 4x - 3y= 6 and having y-intercept -5?

    4x - 3y = 6 के लंबवत रेखा और y-��त�छेद -5 होन ेवाल ेरेखा का समीकरण �या ह?ै"Options:1) 3x + 4y = 202) 3x + 4y = -203) 4x - 3y = -204) 4x - 3y = 20Correct Answer: 3x + 4y = -20Candidate Answer: 3x + 4y = -20 QID : 643 - D and E are points on sides AB and AC of ΔABC. DE is parallel to BC. If AD:DB = 1:2 and area of ΔADE is 3 sq cm, what is the area (in sq cm) ofquadrilateral BDEC?

    ΔABC के AB और AC भुजा� पर D और E दो �ब�� ह�। DE, BC के समानांतर है। य�द AD:DB = 1:2 और ΔADE का �े�फल 3 वग� स.ेमी. ह,ै तो चतुभु�ज BDEC का �े�फल (वग� स.ेमी. म�) �या ह?ैOptions:1) 242) 213) 274) 30Correct Answer: 24Candidate Answer: 24 QID : 644 - What is the value of x?2 Cot 60o - (1/√3) Cos 45o = x

    x का मान �या होगा?2 Cot 60o - (1/√3) Cos 45o = xOptions:1) (3 - 2√2)/3√22) (2√2 - 1)/√63) 7/34) 2Correct Answer: (2√2 - 1)/√6Candidate Answer: (2√2 - 1)/√6 

    QID : 645 - Δ LMN is right angled at M. If m∠N = 45o. What is the length of MN (in cm), if NL = 8 cm?

    Δ LMN, M पर समकोण है। य�द m∠N = 45o ह,ै तो MN क� लंबाई (स.ेमी. म�) �या ह,ै य�द NL = 8 स.ेमी. ह?ैOptions:1) 8√22) 4/√23) 4√24) 4Correct Answer: 4√2Candidate Answer: 4√2 QID : 646 - If Cot θ = 15/8, then Cosec θ = ?

    य�द Cot θ = 15/8, तो Cosec θ = ?

    Downloaded from www.kvclasses.com

    Visit www.kvclasses.com for tons of free study materials and latest exam updates.

    www.

    kvcl

    asse

    s.com

  • Options:1) 8/172) 8/153) 15/174) 17/8Correct Answer: 17/8Candidate Answer: 17/8 QID : 647 -

    The household spends the most for which item?

    गृह�थी �कस व�त ुपर सबसे अ�धक खच� करती ह?ैOptions:1) Rent�कराया2) Travelया�ा3) Food खा�4) Feesशु�क

    Correct Answer: Food खा�

    Candidate Answer: Food खा�

     QID : 648 -

    If the household's monthly income is Rs 50,000, then its savings are ___________ percent of its income.

    य�द एक गृह�थी क� मा�सक आय 50,000 � ह,ै तो उसक� बचत आमदनी का ___________ ��तशत होती है।

    Downloaded from www.kvclasses.com

    Visit www.kvclasses.com for tons of free study materials and latest exam updates.

    www.

    kvcl

    asse

    s.com

  • Options:1) 142) 283) 364) 72Correct Answer: 28Candidate Answer: 28 QID : 649 -

    If the travel expense had been Rs 4000 more, then the ratio of fees to travel would have been?

    य�द या�ा म� 4000 � अ�धक खच� हो जाए,ं तो शु�क और या�ा का अनुपात �कतना होगा?Options:1) 3 : 52) 5 : 33) 3 : 14) 1 : 3Correct Answer: 1 : 3Candidate Answer: 1 : 3 QID : 650 -

    What is the sum of the angular degrees of the sectors representing food and utilities?

    खा� और उपयो�गता� को ��त�न�ध�व करन ेवाल े�े�� के कोणीय �ड�ीय� का योग �कतना होगा?Options:1) 1402) 1503) 1704) 160Correct Answer: 170Candidate Answer: 170 

    Downloaded from www.kvclasses.com

    Visit www.kvclasses.com for tons of free study materials and latest exam updates.

    www.

    kvcl

    asse

    s.com

  • QID : 651 - In the following question, some part of the sentence may have errors. Find out which part of the sentence has an error and select the appropriate option. Ifa sentence is free from error, select 'No Error'.

    Today, it (1)/ has become (2)/ very popular. (3)/ No Error (4)Options:1) 12) 23) 34) 4Correct Answer: 4Candidate Answer: 2 QID : 652 - In the following question, some part of the sentence may have errors. Find out which part of the sentence has an error and select the appropriate option. Ifa sentence is free from error, select 'No Error'.

    They decorated a (1)/ pine tree from (2)/ gleaming bells and baubles. (3)/ No Error (4)Options:1) 12) 23) 34) 4Correct Answer: 2Candidate Answer: 2 QID : 653 - In the following question, a sentence is given with a blank to be filled in with an appropriate word. Select the correct alternative out of the four and indicateit by selecting the appropriate option.

    That was my first glimpse of the __________ Flamingo.Options:1) magnify2) magnificence3) magnification4) magnificentCorrect Answer: magnificentCandidate Answer: magnificent QID : 654 - In the following question, a sentence is given with a blank to be filled in with an appropriate word. Select the correct alternative out of the four and indicateit by selecting the appropriate option.

    We collected _____________ on the design from similar gardens.Options:1) inform2) informal3) informs4) informationCorrect Answer: informationCandidate Answer: information QID : 655 - In the following question, out of the four alternatives, select the word similar in meaning to the word given.

    AbstruseOptions:1) Clear2) Lucid3) Esoteric4) ConcreteCorrect Answer: EsotericCandidate Answer: Esoteric QID : 656 - In the following question, out of the four alternatives, select the word similar in meaning to the word given.

    EminenceOptions:1) Impotence2) Triviality3) Subservience4) RenownCorrect Answer: RenownCandidate Answer: Renown QID : 657 - In the following question, out of the four alternatives, select the word opposite in meaning to the word given.

    Recognition

    Downloaded from www.kvclasses.com

    Visit www.kvclasses.com for tons of free study materials and latest exam updates.

    www.

    kvcl

    asse

    s.com

  • Options:1) Acceptance2) Concession3) Repudiation4) CognizanceCorrect Answer: RepudiationCandidate Answer: Repudiation QID : 658 - In the following question, out of the four alternatives, select the word opposite in meaning to the word given.

    DegenerateOptions:1) Perverted2) Healthy3) Retrograde4) NefariousCorrect Answer: HealthyCandidate Answer: Perverted QID : 659 - In the following question, out of the four alternatives, select the alternative which best expresses the meaning of the idiom/phrase.

    Self-made personOptions:1) A selfish person who will go to any length to harm others for his own benefit2) A person who was born poor but achieves success by hard work and ingenuity3) A person of great emotional strength who can be strong even in the times of crisis4) A person though himself disadvantaged goes out of his way to help othersCorrect Answer: A person who was born poor but achieves success by hard work and ingenuityCandidate Answer: A person who was born poor but achieves success by hard work and ingenuity QID : 660 - In the following question, out of the four alternatives, select the alternative which best expresses the meaning of the idiom/phrase.

    Through thick and thinOptions:1) Lose and gain weight easily2) Difficult times are always followed by easier times3) Big or small, difficulties are difficulties4) In both good and bad timesCorrect Answer: In both good and bad timesCandidate Answer: In both good and bad times QID : 661 - In the following question, out of the four alternatives, select the alternative which will improve the bracketed part of the sentence. In case no improvementis needed, select "no improvement".

    Each student has five plants (will allot) to him / her.Options:1) allot2) allotting3) allotted4) No improvementCorrect Answer: allottedCandidate Answer: allotted QID : 662 - In the following question, out of the four alternatives, select the alternative which will improve the bracketed part of the sentence. In case no improvementis needed, select "no improvement".

    Ravi and his grandfather (were walk) when they saw an injured parrot lying on the ground.Options:1) is walking2) was walking3) were walking4) No improvementCorrect Answer: were walkingCandidate Answer: were walking QID : 663 - In the following question, out of the four alternatives, select the alternative which is the best substitute of the words / sentence.

    Repay a person who has spent or lost moneyOptions:1) Forfeiture2) Amercement3) Reimburse4) MulctCorrect Answer: ReimburseCandidate Answer: Reimburse 

    Downloaded from www.kvclasses.com

    Visit www.kvclasses.com for tons of free study materials and latest exam updates.

    www.

    kvcl

    asse

    s.com

  • QID : 664 - In the following question, out of the four alternatives, select the alternative which is the best substitute of the words / sentence.

    A person who delays or puts things off — like work, chores, or other actionsOptions:1) Diligent2) Procrastinator3) Persevere4) AssiduousCorrect Answer: ProcrastinatorCandidate Answer: Procrastinator QID : 665 - In the following question, four words are given out of which one word is correctly spelt. Select the correctly spelt word.Options:1) millenia2) milennia3) milenia4) millenniaCorrect Answer: millenniaCandidate Answer: milennia QID : 666 - In the following question, four words are given out of which one word is correctly spelt. Select the correctly spelt word.Options:1) cantended2) contended3) contendded4) cantenddedCorrect Answer: contendedCandidate Answer: contended QID : 667 - The question below consists of a set of labelled sentences. Out of the four options given, select the most logical order of the sentences to form a coherentparagraph.

    But their physical concomitantsX-mental phenomena can beY-ordered by a reference to theseZ-have a time and place, andOptions:1) YZX2) ZXY3) YXZ4) XZYCorrect Answer: ZXYCandidate Answer: ZXY QID : 668 - The question below consists of a set of labelled sentences. Out of the four options given, select the most logical order of the sentences to form a coherentparagraph.

    The site is near Russell Glacier, aX-the massive inland ice sheetY-that covers most of the islandZ-dynamic front protruding fromOptions:1) YZX2) YXZ3) ZXY4) XZYCorrect Answer: ZXYCandidate Answer: ZXY QID : 669 - In the following question, a sentence has been given in Active / Passive voice. Out of the four alternatives suggested, select the one which best expressesthe same sentence in Passive / Active voice.

    I have taken him out.Options:1) He has been taken out by me.2) He will be taken out by me.3) Taking him out has been done by me.4) Taking him out is being done by me.Correct Answer: He has been taken out by me.Candidate Answer: He has been taken out by me. QID : 670 - In the following question, a sentence has been given in Direct / Indirect speech. Out of the four alternatives suggested, select the one which bestexpresses the same sentence in Indirect / Direct speech.

    “Maybe you are here to meet me?” the manager said to the visitor.

    Downloaded from www.kvclasses.com

    Visit www.kvclasses.com for tons of free study materials and latest exam updates.

    www.

    kvcl

    asse

    s.com

  • Options:1) The manager told the visitor that maybe he was here to meet him.2) The manager told the visitor that maybe he was there to meet me.3) The manager told the visitor that maybe he was here to meet me.4) The manager told the visitor that maybe he was there to meet him.Correct Answer: The manager told the visitor that maybe he was there to meet him.Candidate Answer: The manager told the visitor that maybe he was there to meet him. QID : 671 - In the following passage, some of the words have been left out. Read the passage carefully and select the correct answer for the given blank out of thefour alternatives.

    In their very early journey through time, languages acquired whole new spectrum of hues and _____________, to describe a character or situation, to add emotion,humour, and to evoke feelings. Figures __________ speech evolved to give the desired effect, mood and so ________. They _________ to the needs of the sculptors oflanguage, such as writers, poets and speakers. Finally, man was getting control over the language rather than language limiting his thought ________ expression.

    new spectrum of hues and _____________, to describe a characterOptions:1) refinements2) refining3) refined4) to refineCorrect Answer: refinementsCandidate Answer: refinements QID : 672 - In the following passage, some of the words have been left out. Read the passage carefully and select the correct answer for the given blank out of thefour alternatives.

    In their very early journey through time, languages acquired whole new spectrum of hues and _____________, to describe a character or situation, to add emotion,humour, and to evoke feelings. Figures __________ speech evolved to give the desired effect, mood and so ________. They _________ to the needs of the sculptors oflanguage, such as writers, poets and speakers. Finally, man was getting control over the language rather than language limiting his thought ________ expression.

    Figures __________ speech evolved to give the desired effectOptions:1) of2) for3) to4) fromCorrect Answer: ofCandidate Answer: of QID : 673 - In the following passage, some of the words have been left out. Read the passage carefully and select the correct answer for the given blank out of thefour alternatives.

    In their very early journey through time, languages acquired whole new spectrum of hues and _____________, to describe a character or situation, to add emotion,humour, and to evoke feelings. Figures __________ speech evolved to give the desired effect, mood and so ________. They _________ to the needs of the sculptors oflanguage, such as writers, poets and speakers. Finally, man was getting control over the language rather than language limiting his thought ________ expression.

    speech evolved to give the desired effect, mood and so ________.Options:1) on2) in3) up4) soCorrect Answer: onCandidate Answer: on QID : 674 - In the following passage, some of the words have been left out. Read the passage carefully and select the correct answer for the given blank out of thefour alternatives.

    In their very early journey through time, languages acquired whole new spectrum of hues and _____________, to describe a character or situation, to add emotion,humour, and to evoke feelings. Figures __________ speech evolved to give the desired effect, mood and so ________. They _________ to the needs of the sculptors oflanguage, such as writers, poets and speakers. Finally, man was getting control over the language rather than language limiting his thought ________ expression.

    They _________ to the needs of the sculptors of languageOptions:1) was catering2) catering3) were catering4) is cateringCorrect Answer: were cateringCandidate Answer: were catering 

    Downloaded from www.kvclasses.com

    Visit www.kvclasses.com for tons of free study materials and latest exam updates.

    www.

    kvcl

    asse

    s.com

  • QID : 675 - In the following passage, some of the words have been left out. Read the passage carefully and select the correct answer for the given blank out of thefour alternatives.

    In their very early journey through time, languages acquired whole new spectrum of hues and _____________, to describe a character or situation, to add emotion,humour, and to evoke feelings. Figures __________ speech evolved to give the desired effect, mood and so ________. They _________ to the needs of the sculptors oflanguage, such as writers, poets and speakers. Finally, man was getting control over the language rather than language limiting his thought ________ expression.

    language limiting his thought ________ expression.Options:1) nor2) neither3) or4) butCorrect Answer: orCandidate Answer: or QID : 676 - Who among the following takes most of the important decisions regarding the savings and loan activities in a Self Help Group?

    �न�न�ल�खत म� स ेकौन �वय ंसहायता समूह म� बचत तथा ऋण ग�त�व�धय� संबंधी अ�धकतर मह�वपूण� �नण�य लेता ह?ैOptions:1) President of Self Help Group�वय ंसहायता समूह का अ�य�2) Leader of Self Help Group�वय ंसहायता समूह का नेता3) Members of Self Help Group�वय ंसहायता समूह के सद�य4) No option is correctकोई �वक�प सही नह� हैCorrect Answer: Members of Self Help Group�वय ंसहायता समूह के सद�यCandidate Answer: Members of Self Help Group�वय ंसहायता समूह के सद�य QID : 677 - Which among the following word is not written in Preamble of our Constitution?

    �न�न�ल�खत म� स ेकौन सा श�द भारतीय सं�वधान के उ�े�शका म� नह� �लखा गया ह?ैOptions:1) Sovereignसंपूण� �भु�व2) Socialistसमाजवाद�3) Secularधम��नरपे�4) Communistसा�यवाद�

    Correct Answer: Communistसा�यवाद�

    Candidate Answer: Secularधम��नरपे�

     QID : 678 - In India, who appoints the Governors of the State?

    भारत म�, रा�य के रा�यपाल� क� �नयु�� कौन करता ह?ैOptions:1) The Supreme Court of Indiaभारत का उ�चतम �यायालय2) The Chief Minister of Stateरा�य के मु�यमं�ी3) The President of Indiaभारत के रा�प�त4) The Chief Justice of Indiaभारत के मु�य �यायाधीशCorrect Answer: The President of Indiaभारत के रा�प�तCandidate Answer: The President of Indiaभारत के रा�प�त QID : 679 - What was the main cause of Civil Disobedience Movement,1930?

    स�वनय अव�ा आंदोलन, 1930 का मु�य कारण �या था?

    Downloaded from www.kvclasses.com

    Visit www.kvclasses.com for tons of free study materials and latest exam updates.

    www.

    kvcl

    asse

    s.com

  • Options:1) Protest against British monopoly on saltनमक पर अं�ेज़ो के एका�धकार के �खलाफ �वरोध2) Compulsion of opium farmingअफ�म खेती क� मजबूरी3) Compulsion of indigo farmingनील खेती क� मजबूरी4) Low labour wagesकाम �म भ�ाCorrect Answer: Protest against British monopoly on saltनमक पर अं�ेज़ो के एका�धकार के �खलाफ �वरोधCandidate Answer: Low labour wagesकाम �म भ�ा QID : 680 - In which year did Mahatma Gandhi gave a call for a Satyagraha against the Rowlatt Act?

    �कस वष� म� महा�मा गांधी न ेरॉलेट कानून के �व�� स�या�ह का आ�ाहन �कया था?Options:1) 19152) 19193) 19234) 1909Correct Answer: 1919Candidate Answer: 1919 QID : 681 - The Iron Pillar located in Quwwat Ul Mosque is in which city? 

    क़� �वत उल म��जद म� ��थत लौह �तंभ �कस शहर म� ह ै?Options:1) Mysoreमैसूर2) Delhi�द�ली3) Hyderabadहैदराबाद4) Kolkataकोलकाता

    Correct Answer: Delhi�द�ली

    Candidate Answer: Delhi�द�ली

     QID : 682 - Which is the smallest state of India in terms of area?

    �े�फल के संदभ� म� भारत का सबसे छोटा रा�य कौन सा ह?ै Options:1) Sikkim�स��कम2) Goaगोवा3) Manipurम�णपुर4) Keralaकेरल

    Correct Answer: Goaगोवा

    Candidate Answer: Goaगोवा

     QID : 683 - Which is the major geologically young division of India that stretches over the northern borders of India?

    भारत का �मुख भूगभ�य युवा �वभाजन कौन सा ह ैजो भारत क� उ�री सीमा� पर फैला ह?ैOptions:1) The Himalayas�हमालय2) The Peninsular Plateau�ाय��प पठार3) The Indian Desertभारतीय रे�ग�तान4) The Northern Plainsउ�री मैदानCorrect Answer: The Himalayas�हमालय

    Candidate Answer: The Himalayas�हमालय

    Downloaded from www.kvclasses.com

    Visit www.kvclasses.com for tons of free study materials and latest exam updates.

    www.

    kvcl

    asse

    s.com

  •  QID : 684 - The western part of the Northern Plains in India is referred to as the ______.

    भारत म� उ�री मैदान� का प��मी भाग ______ के �प म� जाना जाता है।Options:1) The Ganga Plainsगंगा के मैदान2) The Brahmaputra Plains��पु� के मैदान3) The Punjab Plainsपंजाब के मैदान4) No option is correctकोई �वक�प सही नह� हैCorrect Answer: The Punjab Plainsपंजाब के मैदानCandidate Answer: [ NOT ANSWERED ] QID : 685 - Which is the largest inhabited riverine island in the world?

    �व� का सबस ेबड़ा आवासीय नद� ��प कौन सा ह?ैOptions:1) Majuliमाजुली2) Doabदोआब3) Umanandaउमानंद4) Marajoमाराजो

    Correct Answer: Majuliमाजुली

    Candidate Answer: Majuliमाजुली

     QID : 686 - Which state of India is the largest producer of iron ore?

    भारत का कौन सा रा�य लौह अय�क (क�च ेलोह)े का सबसे बड़ा उ�पादक ह?ैOptions:1) Jharkhandझारखंड2) Andhra Pradeshआं� �देश3) Odishaओ�डशा4) West Bengalप��म बंगालCorrect Answer: Odishaओ�डशा

    Candidate Answer: Jharkhandझारखंड

     QID : 687 - The jet streams are located approximately over ______ north latitude in India, therefore, they are called subtropical westerly jet streams.

    जेट धाराए ंभारत म� लगभग ______ उ�री अ�ांश पर ��थत ह�, इस�लए उ�ह� उप-उ�णक�टबंधीय प��मी जेट धारा कहा जाता है।Options:1) 27°-30°2) 20°-23°3) 35°-37°4) 22°-25°Correct Answer: 27°-30°Candidate Answer: 27°-30° QID : 688 - A and B can do a job in 5 days, B and C can do the same job in 6 days and C and A can do the same job in 7.5 days. A, B and C together will complete thesame job in how many days?

    A तथा B �कसी काय� को 5 �दन� म�, B तथा C उसी काय� को 6 �दन� म� तथा C तथा A उसी काय� को 7.5 �दन� म� कर सकत ेह�। A, B तथा C साथ �मलकर उसी काय� को �कतन े�दन� म� पूरा करेग�?Options:1) 82) 43) 24) 6Correct Answer: 4Candidate Answer: 2 

    Downloaded from www.kvclasses.com

    Visit www.kvclasses.com for tons of free study materials and latest exam updates.

    www.

    kvcl

    asse

    s.com

  • QID : 689 - When two or more cells are connected to each other by opposite terminals, then it is called as ______.

    जब दो या अ�धक सेल� को आपस म� उनके �वपरीत ट�म�नल� स ेजोड़ा जाता ह,ै तब उस े______ कहत ेहै।Options:1) electric bulb�व�ुत ब�ब2) switch��वच3) batteryबैटरी4) electric current�व�ुत धाराCorrect Answer: batteryबैटरी

    Candidate Answer: batteryबैटरी

     QID : 690 - Plasma membrane is made up of which organic molecules?

    �ला�मा �झ�ली �कन काब��नक अणु� स ेबनी होती ह?ैOptions:1) Carbohydratesकाब�हाइ�ेट2) Vitamin�वटा�मन3) Lipids and proteinवसा तथा �ोट�न4) Roughage��ांश

    Correct Answer: Lipids and proteinवसा तथा �ोट�नCandidate Answer: Carbohydratesकाब�हाइ�ेट

     QID : 691 - During Galvanization, which metal is used for coating iron and steel objects, preventing them from rusting?

    गै��वनीकरण के दौरान, लोह ेतथा इ�पात को जंग स ेसुर��त रखन ेके �लए �कस धात ुका को�ट�ग के �लए �योग �कया जाता ह?ैOptions:1) Zincज�ता2) Copperताँबा3) Aluminiumअ�यु�म�नयम4) Bronzeकाँसा

    Correct Answer: Zincज�ता

    Candidate Answer: Zincज�ता

     QID : 692 - For whom the Union Ministry of Human Resource and Development has launched ‘Diksha Portal’ for providing digital platform?

    �कसके �लए मानव संसाधन और �वकास मं�ालय न े�ड�जटल मंच �दान करन ेके �लए 'द��ा पोट�ल' शु� �कया ह?ैOptions:1) Teachers�श�क�2) Studentsछा��3) Doctors�च�क�सक�4) Lawyersवक�ल�

    Correct Answer: Teachers�श�क�

    Candidate Answer: Studentsछा��

     QID : 693 - World’s largest combustion research centre has been inaugurated at which Indian institute?

    �कस भारतीय सं�थान म� ��नया के सबसे बड़े दहन अनुसंधान क� � का उ�ाटन �कया गया ह?ै

    Downloaded from www.kvclasses.com

    Visit www.kvclasses.com for tons of free study materials and latest exam updates.

    www.

    kvcl

    asse

    s.com

  • Options:1) IIT Guwahatiआईआईट� गुवाहाट�2) IIT Bombayआईआईट� बॉ�बे3) IIT Delhiआईआईट� �द�ली4) IIT Madrasआईआईट� म�ासCorrect Answer: IIT Madrasआईआईट� म�ासCandidate Answer: [ NOT ANSWERED ] QID : 694 - Which country has won the 2017 FIFA Under-17 World Cup?

    �कस देश न े2017 फ�फा अंडर -17 �व� कप जीता ह?ैOptions:1) Ghanaघाना2) Costa Ricaको�टा �रका3) Englandइं�ल�ड4) Brazil�ाज़ील

    Correct Answer: Englandइं�ल�ड

    Candidate Answer: Englandइं�ल�ड

     QID : 695 - On 20 September 2017, India has announced to build a nuclear plant in ___________, in collaboration with Russia.

    20 �सतंबर 2017 को, भारत न े�स के सहयोग स,े ___________ म� एक परमाण ुसंयं� बनाने क� घोषणा क� है।Options:1) Afghanistanअफगा�न�तान2) Nepalनेपाल3) Bangladeshबां�लादेश4) Myanmar�यांमार

    Correct Answer: Bangladeshबां�लादेश

    Candidate Answer: Myanmar�यांमार

     QID : 696 - Geneva-based ICAN has been awarded with Nobel Peace Prize 2017. ICAN campaigns against  ______.

    �जनेवा आधा�रत आईसीएएन (ICAN) को नोबेल शां�त पुर�कार 2017 स ेस�मा�नत �कया गया है। आईसीएएन (ICAN) ______ के �व�� अ�भयान करता है।Options:1) Nuclear Weaponपरमाण ुह�थयार2) wars between countriesदेश� के बीच यु�3) missiles�मसाइ�स4) submarinesपनडु��बय�

    Correct Answer: Nuclear Weaponपरमाण ुह�थयारCandidate Answer: Nuclear Weaponपरमाण ुह�थयार QID : 697 - Which organisation has been conferred with the ‘Maharatna’ status in September, 2017?

    �सतंबर, 2017 म� �कस संगठन को 'महार�न' का दजा� �दया गया ह?ै

    Downloaded from www.kvclasses.com

    Visit www.kvclasses.com for tons of free study materials and latest exam updates.

    www.

    kvcl

    asse

    s.com

  • Options:1) Hindustan Shipyard Limited (HSL)�ह���तान �शपयाड� �ल�मटेड (एचएसएल)2) Bharat Heavy Plates and Vessels (BHPV)भारत भारी �लेट्स तथा वेस�स (बीएचपीवी)3) Bharat Petroleum Corporation Limited (BPCL)भारत पे�ो�लयम कॉप�रेशन �ल�मटेड (बीपीसीएल)4) Oil and Natural Gas Corporation (ONGC)तेल तथा �ाकृ�तक गैस �नगम (ओएनजीसी)Correct Answer: Bharat Petroleum Corporation Limited (BPCL)भारत पे�ो�लयम कॉप�रेशन �ल�मटेड (बीपीसीएल)Candidate Answer: [ NOT ANSWERED ] QID : 698 - Japan has decided to be associated with the development of which three cities as smart cities in India?

    जापान न ेभारत के �कन तीन शहर� को �माट� शहर� म� �वक�सत करन ेके काय� के साथ जुड़े रहन ेका �नण�य �कया ह?ैOptions:1) Bhopal, Lucknow and Visakhapatnamभोपाल, लखनऊ तथा �वशाखापट्नम2) Chennai, Ahmedabad and Varanasiचे�ई, अहमदाबाद तथा वाराणसी3) Indore, Meerut and Salemइंदौर, मेरठ तथा सेलम4) Chandigarh, Shillong and Nagpurचंडीगढ़, �शलांग तथा नागपुरCorrect Answer: Chennai, Ahmedabad and Varanasiचे�ई, अहमदाबाद तथा वाराणसीCandidate Answer: Indore, Meerut and Salemइंदौर, मेरठ तथा सेलम QID : 699 - India’s first electric bus service to curb carbon emissions has been launched in which state?

    काब�न उ�सज�न को रोकन ेके �लए भारत क� पहली �व�ुत बस सेवा �कस रा�य म� आरंभ क� गई ह?ैOptions:1) Jammu and Kashmirज�म ूऔर क�मीर2) Himachal Pradesh�हमाचल �देश3) Uttarakhandउ�राखंड4) Sikkim�स��कम

    Correct Answer: Himachal Pradesh�हमाचल �देशCandidate Answer: Himachal Pradesh�हमाचल �देश QID : 700 - Who has been appointed as chairman of the Film and Television Institute of India (FTII) in October, 2017?

    अ�टूबर 2017 म�, भारतीय �फ�म तथा टेली�वज़न सं�थान (एफट�आईआई) के अ�य� के �प म� �कस े�नयु� �कया गया ह?ैOptions:1) Anupam Kherअनुपम खेर2) Gajendra Chauhanगज�� चौहान3) Annu Kapoorअ� ूकपूर4) Kamlesh Gillकमलेश �गलCorrect Answer: Anupam Kherअनुपम खेरCandidate Answer: Anupam Kherअनुपम खेर 

    QID : 701 - If two point charges 'q1,' 'q2 'are separated by a distance 'r' in vacuum, the magnitude of the force (F) between them is given by F= q1q2/(4πεor2). Herewhere εo = _________ x 10-12 C2N-1m-2.

    य�द दो �ब�� आवेश 'q1,' 'q2 ' को �नवा�त म� �री r स े�वभा�जत �कया जाता ह,ै तो उनके बीच बल (F) क� ती�ता F= q1q2/(4πεor2). �ारा द� जाती है। यहा ंजहा ंεo = _________ x 10-12 कुल�ब2 �युटन-1मी-2।Options:1) 1.672) 9.113) 8.854) 5.32Correct Answer: 8.85

    Downloaded from www.kvclasses.com

    Visit www.kvclasses.com for tons of free study materials and latest exam updates.

    www.

    kvcl

    asse

    s.com

  • Candidate Answer: 8.85 QID : 702 - "Whenever two (or more) waves travel through the same medium at the same time, the net displacement of the medium at any point in space or time, issimply the sum of the individual wave displacements." This statement is the principle ____________.

    "जब दो (या अ�धक) तरंग े�कसी समान मा�यम म� समान समय पर गमन करती ह� तो कुल �व�थापन ��येक तरंग के कारण होन ेवाल े�व�थापन का योग होता है।" यह कथन _____ �स�ांत का है।Options:1) doppler effectडॉ�लर �भाव2) resonanceअनुनाद3) superpositionअ�यारोपण4) harmonicsआवृ��

    Correct Answer: superpositionअ�यारोपण

    Candidate Answer: superpositionअ�यारोपण

     QID : 703 - "Every body continues to be in its state of rest or of uniform motion in a straight line unless compelled by some external force to act otherwise." Thisstatement is ____________.

    "��येक �प�ड तब तक अपनी �वरामाव�था अथवा सरल रेखा म� एकसमान ग�त क� अव�था म� रहता ह ैजब तक कोई बा� बल उस ेअ�यथा �वहार करन ेके �लए �ववश नह� करता।" यह कथन ______ है।Options:1) Newton's 1st Law�यूटन का �थम �नयम2) Newton's 2nd Law�यूटन का ��तीय �नयम3) Law of conservation of energyउजा� संर�ण का �नयम4) Newton's 3rd Law�यूटन का तृतीय �नयम

    Correct Answer: Newton's 1st Law�यूटन का �थम �नयम

    Candidate Answer: Newton's 1st Law�यूटन का �थम �नयम 

    QID : 704 - [M0L2T-2] are the dimensions of _______.

    [M0L2T-2] ______ क� �वमाय� ह�।Options:1) Gravitational potential energyगु��वीय ��थ�तज उजा�2) Gravitational potentialगु��वीय �वभव3) Gravitational constantगु��वीय ��थरांक4) Gravitational intensityगु��वीय ती�ताCorrect Answer: Gravitational potentialगु��वीय �वभवCandidate Answer: Gravitational constantगु��वीय ��थरांक QID : 705 - ___________ is a device by which a system is made to undergo a cyclic process that results in conversion of heat to work.

    ______ एक ऐसी यु�� ह ै�जसम� �नकाय �ारा च��य ��म पूरा कराया जाता ह ै�जसके फल�व�प ऊ�मा काय� म� �पांत�रत होती है।Options:1) Refrigerator�शीतक2) Heat engineऊ�मा इंजन3) Heat pumpऊ�मा पंप4) Heat Sinkऊ�मा �स�कCorrect Answer: Heat engineऊ�मा इंजनCandidate Answer: Heat engineऊ�मा इंजन 

    Downloaded from www.kvclasses.com

    Visit www.kvclasses.com for tons of free study materials and latest exam updates.

    www.

    kvcl

    asse

    s.com

  • QID : 706 -

    Options:1) A-3, B-2, C-12) A-2, B-3, C-13) A-1, B-2, C-34) A-2, B-1, C-3Correct Answer: A-2, B-3, C-1Candidate Answer: A-2, B-3, C-1 

    QID : 707 - Which of these materials has the highest conductivity at 0o C?

    0 �ड�ी से��सयस पर �न�न�ल�खत म� स े�कस पदाथ� क� चालकता सबसे अ�धक होती ह?ैOptions:1) Ironलोहा2) Graphite�ेफ़ाइट3) Glassकांच4) Waterजल

    Correct Answer: Ironलोहा

    Candidate Answer: Graphite�ेफ़ाइट

     QID : 708 - 1 decay per second equals 1 ______________.

    1 �य ��त सेक�ड 1 ____ के बराबर होता है।Options:1) curie�यूरी2) gray�े3) becquerelबैकेरल4) sievertसीएवेट�

    Correct Answer: becquerelबैकेरल

    Candidate Answer: becquerelबैकेरल

     QID : 709 - 1 torr = _________ Pa.

    1 टॉर = ____ Pa।Options:1) 760

    2) 1.013

    3) 100000

    4) 133Correct Answer: 133Candidate Answer: 760 

    Downloaded from www.kvclasses.com

    Visit www.kvclasses.com for tons of free study materials and latest exam updates.

    www.

    kvcl

    asse

    s.com

  • QID : 710 - A thin ring of mass 10 kg is rolling on horizontal ground such that its centre of mass has a velocity of 2 m/s. How much work (in joules) needs to be doneto stop it?

    10 �क.�ा. ��मान वाला एक पतला छ�ला �ै�तज फश� पर इस �कार लोट�नक ग�त करता ह ै�क इसके ��मान क� � क� चाल 2 मी./सेक�ड हो। इस ेरोकन ेके �लए �कतना काय� करना होगा? (जूल म�)Options:1) 402) 203) 104) 80Correct Answer: 40Candidate Answer: 20 QID : 711 - What would be the total power (in diopters) of a combination of a convex lens of focal length 10 cm and a concave lens of focal length 15 cm?

    10 स�.मी. फोकस �री वाल ेउ�ल ल�स और 15 स�.मी. फोकस �री वाल ेअवतल ल�स के संयोजन क� कुल �मता (डाइऑ�टर म�) �कतनी होगी?Options:1) -3.332) 53) 3.334) -5Correct Answer: 3.33Candidate Answer: 3.33 

    QID : 712 - 1n0 + 235U92 → 133Sb51 + 99Nb41 + y1n0. Find ' y ' from the given fission equation of an uranium nucleus?

    1n0 + 235U92 → 133Sb51 + 99Nb41 + y1n0. यूरे�नयम ना�भक के इस �वखंडन समीकरण स े' y ' �ात कर�?Options:1) 32) 23) 44) 1Correct Answer: 4Candidate Answer: 4 

    QID : 713 - 20 oC equals ______________.

    20 ०C _______ के बराबर है।Options:1) 168o Fahrenheit168o फैरनहाइट2) 253.15 K253.15 K3) 313.15 K313.15 K4) 68o Fahrenheit68o फैरनहाइट

    Correct Answer: 68o Fahrenheit68o फैरनहाइट

    Candidate Answer: 68o Fahrenheit68o फैरनहाइट QID : 714 - How much of 1 gm of Astanine-210 (in gm) will be left in 24 hours if half life of Astanine-210 is 8 hours?

    24 घंटे बाद 1 �ाम अ�टा�नन-210 (�ाम म�) क� �कतनी मा�ा बचेगी य�द अ�टा�नन-210 का अ�� जीवन काल 8 वष� ह?ैOptions:1) 0.125

    2) 0.25

    3) 0.0625

    4) 0.03125Correct Answer: 0.125Candidate Answer: 0.125 

    QID : 715 - 2H1 + 2H1 → 3He2 + ___________ + 3.27 Mev. Find the missing element/s.

    2H1 + 2H1 → 3He2 + ___________ + 3.27 Mev। गायब त�व खोज�।

    Downloaded from www.kvclasses.com

    Visit www.kvclasses.com for tons of free study materials and latest exam updates.

    www.

    kvcl

    asse

    s.com

  • Options:1) 1 neutron1 �यू�ॉन2) 1 proton1 �ोटॉन3) 1 electron1 इले��ॉन4) 1 positron1 पॉजी�ॉनCorrect Answer: 1 neutron1 �यू�ॉनCandidate Answer: 1 neutron1 �यू�ॉन QID : 716 -

    Options:1) 1252) 753) 304) 50Correct Answer: 30Candidate Answer: 30 QID : 717 - Each mode of vibration for a molecule like CO contributes _________ to the energy of a molecule.

    CO जैस ेअण ुके �लए कंपन का ��येक �प ____ उजा� का योगदान देती है।Options:1) 2kBT2) kBT/43) kBT/24) kBTCorrect Answer: kBTCandidate Answer: kBT QID : 718 - What is the Moment of Inertia of a solid sphere of mass ' M ' and radius ' R ' about its diameter?

     ' M ' ��मान और ' R ' ���या वाल ेठोस गोल ेका �ास के प�रत: जड�व आघूण� �या होगा?Options:1) MR2/52) MR2/43) 2MR2/54) MR2/3

    Correct Answer: 2MR2/5

    Candidate Answer: 2MR2/5 QID : 719 - A p-type semiconductor is obtained if we dope ________________.

    p-�कार का अ�� चालक तब बनता ह,ै जब ___ अप�म��त क� जाती ह�।

    Downloaded from www.kvclasses.com

    Visit www.kvclasses.com for tons of free study materials and latest exam updates.

    www.

    kvcl

    asse

    s.com

  • Options:1) Silicon with a pentavalent element�स�लकान �कसी पंचसंयोजी त�व के साथ2) Silicon with a trivalent element�स�लकान �कसी ��संयोजी त�व के साथ3) a pentavalent element with Siliconपंचसंयोजी त�व �कसी �स�लकान के साथ4) a trivalent element with Silicon��संयोजी त�व �कसी �स�लकान के साथCorrect Answer: Silicon with a trivalent element�स�लकान �कसी ��संयोजी त�व के साथCandidate Answer: a trivalent element with Silicon��संयोजी त�व �कसी �स�लकान के साथ QID : 720 - The resonant frequency of a RLC circuit is equal to __________.

    RLC प�रपथ क� अनुनाद आवृ�� _____ के बराबर होती है।Options:1) 1/(LC)2) 1/(LC)23) √(LC)4) 1/√(LC)Correct Answer: 1/√(LC)Candidate Answer: 1/√(LC) QID : 721 - For paramagnetic materials, magnetic susceptibility (χ) is _______________.

    अनुचुंबक�य पदाथ� के �लए चुंबक�य �वृ�� (χ)                             होती है।Options:1) positive and smallधना�मक और कम2) negative and smallऋणा�मक और कम3) negative and largeऋणा�मक और अ�धक4) positive and largeधना�मक और अ�धकCorrect Answer: positive and smallधना�मक और कमCandidate Answer: positive and smallधना�मक और कम QID : 722 - A ray of light passing through the first principal focus of a convex lens ___________________ after refraction.

    अवतल ल�स म� ल�स के �थम मु�य फोकस स ेगुजरन ेवाली �काश �करण अपवत�न के प�ात                                हो जाती है।Options:1) emerges without any deviation�बना �कसी �वचलन के �नग�त हो जाती है2) emerges parallel to the principal axisमु�य अ� के समानांतर हो जाती है3) passes through the second principal focus��तीय मु�य फोकस स ेगुज़रती है4) passes through the center of curvatureव�ता ���या के क� � स ेगुज़रती हैCorrect Answer: emerges parallel to the principal axisमु�य अ� के समानांतर हो जाती हैCandidate Answer: emerges parallel to the principal axisमु�य अ� के समानांतर हो जाती है QID : 723 - Fluids offer resistance to motion due to internal friction, this property is called ______________.

    आंत�रक घष�ण के कारण तरल ग�त म� कुछ ��तरोध �दखाते ह�, इस गुण को                                   कहा जाता है।Options:1) viscosity�यानता2) buoyancyउछाल3) specific gravity�व�श� गु��वाकष�ण4) continuityसांत�य

    Correct Answer: viscosity�यानता

    Candidate Answer: viscosity�यानता

    Downloaded from www.kvclasses.com

    Visit www.kvclasses.com for tons of free study materials and latest exam updates.

    www.

    kvcl

    asse

    s.com

  •  QID : 724 - The collector side of a transistor is __________________ in size as compared to the emitter.

    उ�सज�क क� तुलना म� �ां�ज�टर का सं�ाहक                                     साइज़ का होता है।Options:1) moderately doped and largerसाधारण अप�म��त और बड़ा2) moderately doped and smallerसाधारण अप�म��त और छोटा3) very heavily doped and largerब�त अ�य�धक अप�म��त और बड़ा4) very heavily doped and smallerब�त अ�य�धक अप�म��त और छोटाCorrect Answer: moderately doped and largerसाधारण अप�म��त और बड़ाCandidate Answer: [ NOT ANSWERED ] QID : 725 - The ____________ of thermodynamics states that, no process is possible whose sole result is the transfer of heat from a colder object to a hotter object.

    उ�माग�तक� के ___                   के अनुसार, ऐसा कोई भी ��म संभव नह� ह ै�जसका एकमा� प�रमाण �कसी ठंडे �प�ड स े�कसी गम� �प�ड म� ऊ�मा �थानांतरण हो।Options:1) First Law�थम �नयम2) Zeroth Lawशू�य को�ट का �नयम3) Second Law (Clausius statement)��तीय �नयम (�लॉ�शयस का कथन)4) Second Law (Kelvin-Planck statement)��तीय �नयम (के��वन - �लांक का कथन)Correct Answer: Second Law (Clausius statement)��तीय �नयम (�लॉ�शयस का कथन)Candidate Answer: Second Law (Kelvin-Planck statement)��तीय �नयम (के��वन - �लांक का कथन) QID : 726 - A car increases its velocity from 10 m/s to 20 m/s in 5 seconds. Find its acceleration (in m/s²)?

    एक कार का वेग 5 सेक�ड म� 10 मी. /सेकंड स ेबढ़कर 20 मी. /सेकंड हो जाता है। इसका �वरण �ात कर� (मी ./वग� सेकंड म�)?Options:1) 52) 43) 24) 6Correct Answer: 2Candidate Answer: 2 

    QID : 727 - A refrigerator operates between -3 oC and T oC. If its coefficient of performance is 9, then find T?

    एक �शीतक -3 ०C और T ०C के बीच काम करता है। य�द इसका काय�-�न�पादन गुणांक 9 ह ैतो T �ात कर�?Options:1) 3002) 273) 1274) 200Correct Answer: 27Candidate Answer: 27 QID : 728 - A free neutron, unlike a free proton, is unstable. It decays into a proton, an electron and a/an ________________.

    एक मु� �ोटॉन के �वपरीत एक मु� �यू�ॉन अ�थायी होता है। यह एक �ो�ॉन, एक इले��ॉन और एक _____ के �प म� ��यत हो जाता है।Options:1) antineutrino��त�यू��नो2) positronपॉ�ज़�ान3) mesonम�याणु4) muon�युऑन

    Correct Answer: antineutrino��त�यू��नो

    Candidate Answer: antineutrino��त�यू��नो

     

    Downloaded from www.kvclasses.com

    Visit www.kvclasses.com for tons of free study materials and latest exam updates.

    www.

    kvcl

    asse

    s.com

  • QID : 729 - The electric field due to an infinite thin plane sheet of uniform surface charge density ' σ ' is given as ____________. (ϵo is permittivity of free space)

    एकसमान पृ�ीय आवेश घन�व ' σ ' क� पतली अनंत समतल चादर के कारण �व�ुत �े� ____ होता है। (ϵo मु� �े� क� �व�ुतशीलता ह)ैOptions:1) E = σ/(2ϵo)2) E = σ/(2πϵo)3) E = σ/(4ϵo)4) E = σ/(4πϵo)Correct Answer: E = σ/(2ϵo)Candidate Answer: E = σ/(2ϵo) QID : 730 - In which Thermodynamic process is volume constant?

    �कस ऊ�माग�तक�य ��म म� आयतन ��थर रहता ह?ैOptions:1) Isochoricसमआयत�नक2) Isobaricसमदाबीय3) Isothermalसमतापीय4) Adiabatic��ो�म

    Correct Answer: Isochoricसमआयत�नक

    Candidate Answer: Isochoricसमआयत�नक

     QID : 731 - In whose theory the first postulate states that an electron in an atom could revolve in certain stable orbits without the emission of radiant energy?

    �कसके �स�ांत के अ�भगृहीत के अनुसार इले��ॉन �बना �व�करण उजा� के उ�सज�न के �न��त क�ा� म� प�र�मण करत ेह�?Options:1) Planck’s�लांक2) Rydberg's�रडबग�3) Thomson’sटॉमसन4) Bohr'sबोर

    Correct Answer: Bohr'sबोर

    Candidate Answer: Bohr'sबोर

     QID : 732 - In a uniform magnetic field ' B,' a charge 'q' executes a/an __________ orbit in a plane normal to ' B '.

    �कसी एकसमान चुंबक�य �े� ' B ' म� कोई आवेश 'q' ,' B ' के अ�भलंबवत तल म� _____ क�ा म� ग�तमान होता है।Options:1) ellipticalद�घ�वृ�ाकार2) parabolicपरवल�यक3) hyperbolicअ�तपरवल�यक4) circularवृ�ाकार

    Correct Answer: circularवृ�ाकार

    Candidate Answer: circularवृ�ाकार

     QID : 733 - There is an opening of 5 cm² at the bottom of a tank. If the height of water in the tank is 5 m, then at what rate (in m³/s) will water flow out from thisopening? (use g = 10 m/s²)

    �कसी टंक� क� तली म� 5 वग� स�.मी. का �छ� है। य�द टंक� म� पानी क� ऊंचाई 5 मी. ह ैतो पानी �कस दर (घन मी. /सेकंड म�) स ेइस �छ� स ेबाहर �नकलेगा? (g = 10 मी./वग� सेकंड)Options:1) 5 x 10-4

    2) 2.5 x 10-3

    3) 2.5 x 10-4

    4) 5 x 10-3

    Correct Answer: 5 x 10-3

    Candidate Answer: 2.5 x 10-3

    Downloaded from www.kvclasses.com

    Visit www.kvclasses.com for tons of free study materials and latest exam updates.

    www.

    kvcl

    asse

    s.com

  •  QID : 734 - On reflection from a rigid wall if an incident wave in a string is represented by "y(x, t) = a sin (kx – ωt)," then the amplitude of the stationary wave formed is____________.

    �कसी डोरी म� सु�ढ़ प�रसीमा स ेपराव�त�त होन ेवाली आप�तत तरंग को "y(x, t) = a sin (kx – ωt)" �ारा �द�श�त �कया जाता ह ैतो अ�गामी तरंग� का आयाम _____ होता है।Options:1) A sin(kx)2) A/2 sin(kx)3) 2A sin(kx)4) 2A sin2(kx)Correct Answer: 2A sin(kx)Candidate Answer: A/2 sin(kx) QID : 735 - For a liquid flowing in a pipe where cross section area changes from A1 to A2 and velocity of flow changes from v1 to v2, then A1 x v1 = A2 x v2 is called___________________.

    �कसी पाइप म� �वा�हत होन ेवाल ेतरल के �लए जहा ंअनु��थ प�र�छेद का �े�फल A1 स ेA2 और वेग v1 स ेv2 प�रव�त�त हो जाता ह ैतब A1 x v1 = A2 x v2 को ____ कहा जाता है।Options:1) Equation of conservation of energyउजा� संर�ण का समीकरण2) Equation of magnificationआवध�न का समीकरण3) Equation of translationअंतरण का समीकरण4) Equation of Continuityसांत�य समीकरणCorrect Answer: Equation of Continuityसांत�य समीकरणCandidate Answer: Equation of Continuityसांत�य समीकरण QID : 736 - The property of a body, by virtue of which it tends to regain its original size and shape when the applied force is removed, is known as ___________.

    �कसी �प�ड का वह गुण �जससे वह ��यारो�पत बल को हटान ेपर अपनी �ारं�भक आकृ�त और आकार को पुन: �ा�त कर लेता ह,ै ____ कहलाता है।Options:1) elasticity��या�थता2) plasticity�ला��टकता3) viscosity�यानता4) rigidityकठोरता

    Correct Answer: elasticity��या�थता

    Candidate Answer: elasticity��या�थता

     QID : 737 - The magnitude of the magnetic field at a distance ' R ' from a long, straight wire carrying a current ' I ' is equal to ____________. ( 'µ0 ' is permeability of freespace)

    �कसी लंब ेसीध ेतार �जससे ' I ' �व�ुत धारा �वा�हत हो रही ह,ै स े' R ' �री पर ��थत �कसी �ब�� पर चु�बक�य �े� का प�रमाण ____ के बराबर होता है। ( 'µ0 ' �नवा�त क� चुंबकशीलता है।)Options:1) µoI/(πR2)2) 2µoI/R3) µoI/(2πR)4) µoI/R2

    Correct Answer: µoI/(2πR)Candidate Answer: µoI/(2πR) QID : 738 - The electric energy density (energy per unit volume) in a region with electric field 'E' is ____________ where, ϵo is permittivity of free space.

    �कसी �व�ुत �े� 'E' के �थान पर वै�ुत आवेश घन�व (��त एकांक आयतन उजा�) _____ होता ह ैजहा,ं ϵo �नवा�त म� �व�ुतशीलता है।Options:1) ϵoE/22) ϵo2E/23) 2E2/ϵo4) ϵoE2/2

    Correct Answer: ϵoE2/2

    Candidate Answer: ϵoE2/2 

    Downloaded from www.kvclasses.com

    Visit www.kvclasses.com for tons of free study materials and latest exam updates.

    www.

    kvcl

    asse

    s.com

  • QID : 739 - The ice point of water in Kelvin scale is ________.

    के��वन माप�म पर पानी का �हमांक ___ है।Options:1) 273.15 K

    2) 373.15 K

    3) 0 K

    4) -273.15 KCorrect Answer: 273.15 KCandidate Answer: 273.15 K QID : 740 - An object is placed at 10 cm in front of a concave mirror of radius of curvature 25 cm. Find the position of the image (in cm)?

    कोई व�त ुअवतल दप�ण के सामन े10 स�.मी. क� �री पर रखी है। अवतल दप�ण क� व�ता ���या 25 स�.मी. ह ैतो ��त�ब�ब क� ��थ�त �ात कर� (स�.मी. म�)?Options:1) -502) 403) 504) -40Correct Answer: 50Candidate Answer: 50 QID : 741 -

    Options:1) NAND2) AND3) OR4) NORCorrect Answer: NANDCandidate Answer: NAND QID : 742 - The acceleration due to gravity at a depth ' d ' below the Earth’s surface is given by the formula ______________.

    गु��वाकष�ण के कारण पृ�वी क� पृ� स े' d ' गहराई म� अनुभव �कय ेजान े�वरण का सू� ____ है।Options:1) g = GM x (1-2d/R)/R²2) g = GM x (1-d/R)/R3) g = GM x (1-d/R)/R²4) g = GM x (1-2d/R)/RCorrect Answer: g = GM x (1-d/R)/R²Candidate Answer: g = GM x (1-d/R)/R² QID : 743 - The angle of deflection shown by a galvanometer is inversely proportional to __________________.

    गै�वेनोमीटर का कोणीय �व�ेप ___________ के �ु��मानुपाती होता है।

    Downloaded from www.kvclasses.com

    Visit www.kvclasses.com for tons of free study materials and latest exam updates.

    www.

    kvcl

    asse

    s.com

  • Options:1) number of turns of the coilकंुडली म� फेर� क� सं�या2) cross sectional area of the coilकंुडली का अनु��थ प�र�छेद3) current in the coilकंुडली म� धारा4) torsional constant of the springकमानी का �ठन �नयतांकCorrect Answer: torsional constant of the springकमानी का �ठन �नयतांकCandidate Answer: current in the coilकंुडली म� धारा 

    QID : 744 - In the relation from Kinetic theory of gases, "PV = (1/3)nMv2 ", if ' n ' is number of moles of the gas then ' M ' stands for _________________.

    गैस� क� ग�तक उजा� के संदभ� म� "PV = (1/3)nMv2 " य�द ' n ' गैस के अणु� क� सं�या ह ैतो ' M '                      �द�श�त करता है।Options:1) molecular massआण�वक ��मान2) mass of the particleकण का ��मान3) mass of the gasगैस का ��मान4) number of moles of the gasगैस के अणु� क� सं�याCorrect Answer: molecular massआण�वक ��मानCandidate Answer: molecular massआण�वक ��मान QID : 745 - Which of the following statements is true about density?

    घन�व के संबंध म� कौन सा कथन स�य ह?ैOptions:1) Unit of relative density is kg/m3.सापे� घन�व क� इकाई �क.�ा./मी.3 है।2) Liquids are easily compressible and hence it's density is highly variable.�व आसानी स ेसंपी� होत ेह ैऔर इस�लए इनका घन�व ब�त अचर होता है।3) Gases exhibit a large variation in densities with pressure.गैस दाब म� प�रवत�न के साथ घन�व म� अ�य�धक प�रवत�न दशा�ती ह�।4) Density is a vector quantity.घन�व एक स�दश रा�श है।Correct Answer: Gases exhibit a large variation in densities with pressure.गैस दाब म� प�रवत�न के साथ घन�व म� अ�य�धक प�रवत�न दशा�ती ह�।Candidate Answer: Gases exhibit a large variation in densities with pressure.गैस दाब म� प�रवत�न के साथ घन�व म� अ�य�धक प�रवत�न दशा�ती ह�। QID : 746 - The total force on a charge ' q ' moving with velocity ' v ' in the presence of magnetic and electric fields ' B ' and ' E ' respectively is called the ___________force.

    चु�बक�य �े� ' B ' पर �व�ुत �े� ' E ' क� उप��थ�त म� ' v ' वेग स ेग�तमान �कसी आवेश ' q ' पर लगन ेवाल ेबल को ____ बल कहत ेह�।Options:1) Ampere'sए��पयर2) Biot-Savartबायो सावट�3) Lorentzलोर�ज4) Kirchhoff’s�करचोफ

    Correct Answer: Lorentzलोर�ज

    Candidate Answer: Lorentzलोर�ज

     QID : 747 - When a beam of white light incident on a prism undergoes dispersion then the __________________ of the constituent colors of white light.

    जब कोई सफ़ेद �काश पुंज कांच के ���म पर आप�तत होता ह ैतब इसके संघटक वण� म� _____ ।

    Downloaded from www.kvclasses.com

    Visit www.kvclasses.com for tons of free study materials and latest exam updates.

    www.

    kvcl

    asse

    s.com

  • Options:1) red colour bends the most as it has the lowest wavelengthलाल रंग सबसे अ�धक बंकन होता ह ै�य��क इसक� तरंगदै�य� सबसे कम होती है2) violet colour bends the most as it has the lowest wavelengthब�गनी रंग का बंकन सबसे अ�धक होता ह ै�य��क इसक� तरंगदै�य� सबसे कम होती है3) violet colour bends the most as it has the highest wavelengthब�गनी रंग का बंकन सबसे अ�धक होता ह ै�य��क इसक� तरंगदै�य� सबसे अ�धक होती है4) violet colour bends the most as it has the lowest frequencyब�गनी रंग का बंकन सबसे अ�धक होता ह ै�य��क इसक� आवृ�� सबसे कम होती हैCorrect Answer: violet colour bends the most as it has the lowest wavelengthब�गनी रंग का बंकन सबसे अ�धक होता ह ै�य��क इसक� तरंगदै�य� सबसे कम होती हैCandidate Answer: violet colour bends the most as it has the lowest wavelengthब�गनी रंग का बंकन सबसे अ�धक होता ह ै�य��क इसक� तरंगदै�य� सबसे कम होती है QID : 748 - For a transistor in common emitter configuration ___________________ is the ratio of the change in collector current to the change in base current at aconstant collector-emitter voltage (VCE) when the transistor is in active state.

    �ां�ज�टर के उभ�न� उ�सज�क �व�यास म� �नयत सं�ाहक-उ�सज�क वो�टता (VCE) पर सं�ाहक धारा म� प�रवत�न और आधार धारा म� प�रणामी प�रवत�न के अनुपात को धारा �वध�क गुणांक ____ कहत ेह�।Options:1) Current amplification factorधारा �वध�क गुणांक2) Output resistance�नग�त ��तरोध3) Input resistance�नवेश ��तरोध4) Voltage gainवो�टता ल��धCorrect Answer: Current amplification factorधारा �वध�क गुणांकCandidate Answer: Current amplification factorधारा �वध�क गुणांक QID : 749 - The propagation constant or the angular wave number is equal to ____________.

    तरंग संचरण ��थरांक या कोणीय तरंग सं�या ____________ के बराबर होती है।Options:1) 2πλ2) λ/2π3) 2π/λ4) 1/(2πλ)Correct Answer: 2π/λCandidate Answer: 2π/λ QID : 750 - The relation showing r.m.s. speed (v) of ideal gas particles in terms of temperature is ____________.

    तापमान के संदभ� म� आदश� गैस के कण� के r.m.s. वेग (v) म� यह संबंध ____ होता है।Options:1) v2 = 3kBT/m2) v2 = 3Tm/kB3) v2 = 3kBm/T4) v2 = 3kBTm

    Correct Answer: v2 = 3kBT/m

    Candidate Answer: v2 = 3kBT/m QID : 751 - CP/CV for a rigid diatomic gas is ____________.

    �ढ़ ��परमाणुक गैस के �लए CP/CV = _____।Options:1) 5/72) 7/53) 3/24) 2/3Correct Answer: 7/5Candidate Answer: 7/5 QID : 752 - The 2nd overtone wavelength of a stationary wave formed in a pipe of length 'L' and open at both ends is ____________.

    दोन� �सर� पर खुल ेऔर 'L' ल�बाई के पाइप म� बनने वाली �कसी अ�गामी तरंग म� ��तीय गुणावृ�� ____ होती है।

    Downloaded from www.kvclasses.com

    Visit www.kvclasses.com for tons of free study materials and latest exam updates.

    www.

    kvcl

    asse

    s.com

  • Options:1) L/3

    2) L/2

    3) 2L/3

    4) L/4Correct Answer: 2L/3Candidate Answer: 2L/3 QID : 753 - A diatomic molecule has _________ rotational degrees of freedom.

    ��परमाणुक अण ुक� घूण� �वातं� को�ट ____ होती है।Options:1) twoदो2) threeतीन3) zeroशू�य4) oneएक

    Correct Answer: twoदो

    Candidate Answer: zeroशू�य

     QID : 754 - Which of the following electromagnetic waves has the highest wavelength?

    �न�न�ल�खत म� स े�कन वै�ुतचुंबक�य तरंग� क� तरंगदै�य� सबसे अ�धक होती ह?ैOptions:1) Microwavesसू�म तरंगे2) Radio wavesरे�डयो तरंगे3) Infra-red lightअवर� �काश4) Gamma raysगामा �करण�Correct Answer: Radio wavesरे�डयो तरंगेCandidate Answer: Radio wavesरे�डयो तरंगे QID : 755 - Which of the following has the highest refractive index?

    �न�न�ल�खत म� स े�कसका अपवत�नांक सबसे अ�धक होता ह?ैOptions:1) Waterजल2) Glassकांच3) Vacuum�नवा�त4) Diamondहीरा

    Correct Answer: Diamondहीरा

    Candidate Answer: Diamondहीरा

     QID : 756 - Which of the following is a non-conservative force?

    �न�न�ल�खत म� स ेकौन सा बल असंर�ी बल ह?ैOptions:1) Electrostatic force��थरवै�ुत बल2) Gravitational forceगु���